Q10

 
aaronwfrank
Thanks Received: 2
Vinny Gambini
Vinny Gambini
 
Posts: 23
Joined: August 24th, 2016
 
 
 

Q10

by aaronwfrank Tue Apr 18, 2017 6:30 pm

E is the correct answer. It is the only AC supported by the passage.